Invariancia de Lorentz de un tensor dieléctrico dependiente de la frecuencia y la longitud de onda

Supongamos que tenemos un material descrito por un tensor dieléctrico ϵ ¯ . En el dominio de la frecuencia, este tensor depende de la frecuencia de la onda ω y el vector de onda k .

Claramente no todos ϵ ¯ = F ( ω , k ) son físicamente posibles. Sin considerar la física detrás de cualquier particular ϵ ¯ = F ( ω , k ) , ¿es posible encontrar condiciones generales que todos ϵ ¯ = F ( ω , k ) ¿debe obedecer? Estoy principalmente interesado en la invariancia de Lorenz: seguramente la descripción más general de cualquier material debe estar parametrizada por la velocidad del material según el observador ( v metro a t ), y entonces debe haber algún tipo de simetría entre ω y k en ϵ ¯ = F ( ω , k , v metro a t ) porque la transformación de Lorentz mezcla distancias ( 1 / k ) e intervalos de tiempo ( 1 / ω ).

Pregunta casi idéntica, pero con un título mucho más específico: physics.stackexchange.com/questions/4147/…

Respuestas (1)

Obtiene restricciones sobre la posible forma del dieléctrico ya que es una "función de respuesta". la polarización PAG en un campo aplicado mi es dado por PAG ( X , t ) = ϵ ( X X , t t ) mi ( X , t ) d X d t hasta varias convenciones. Debemos tener ϵ ( X , t ) = 0 cuando t < 0 , ya que un campo eléctrico no puede cambiar cuál era la polarización antes de que se encendiera el campo eléctrico. En el espacio de Fourier esto se traduce en ϵ ( ω ) es analítico en el semicomplejo superior ω avión. Esto a su vez conduce a algo llamado relaciones Kramers-Kronig que puede buscar en wikipedia con ese nombre.

(Supongo que podemos obtener una condición aún más estricta ya que ϵ ( X , t ) debe ser cero fuera del cono de luz delantero, t > | X | .)

En cuanto a ver cómo cambia el dieléctrico con la transformación de Lorentz, esta debería ser una tarea relativamente sencilla. Escriba la ecuación de Maxwell con todo lo expresado en términos de los campos y el dieléctrico y la permitividad y realice una transformación de Lorentz. Esto debería llevarlo de regreso a la misma forma de ecuaciones pero con un dieléctrico diferente.

Puede ser sencillo, pero no se hace en ninguna parte. Obtiene un tensor de respuesta de cuatro índices, y necesita dar una descomposición de simetría, y es molesto (pero no difícil). Esto respondería a un montón de problemas votados en este sitio.